GMAT Data Sufficiency Discussions

The GMAT and Related Discussions section is very fragmented in terms of the information available at one’s disposal. There are one of reply threads and several other redundant threads that have masked the more important ones. This is an atte…

The GMAT and Related Discussions section is very fragmented in terms of the information available at one's disposal. There are one of reply threads and several other redundant threads that have masked the more important ones.

This is an attempt to make the best of explanations, reasonings, questions etc etc easily available to the junta here. We would be having separate threads for topics such as

Problem Solving (the existing one is renamed)

Data Sufficieny
Sentence Correction
Critical Reasoning
Reading Comprehension.

Please ensure that Q & As are carried out in the repsective threads.

Thread is made sticky

PS:-This post would be edited by the MODS in order to give links to various DSs that would be discussed here. We will try to organise the links for the questions discussed here.
Other relevant posts in the 'one of reply' threads would be moved here


Q If xyz 0, is x (y + z) 0?

(1) y + z = |y| + |z
(2) x + y| = |x| + y

Q If xyz 0, is x (y + z) 0?

(1) y + z = |y| + |z
(2) x + y| = |x| + y


Ans is C. Am i correct?
From (1) we can infer y and z have same sign. Similarly from (2) x and y have same signs. So combining both x, y, z have same sign. So x(y+z) is greater than 0.
Ans is C. Am i correct?
From (1) we can infer y and z have same sign. Similarly from (2) x and y have same signs. So combining both x, y, z have same sign. So x(y+z) is greater than 0.


Correct and well explained

Here is one more that's posted in one of threads in this forum

How many different prime factors does N have?

(1) 2N has 4 different prime factors.
(2) N ^2 has 4 different prime factors.

is it B?
1) If N already has 2 a factor then it will have 4 factors but 2 is not a factor then N would have 3 factor. So number of factors is uncertain
2) Squaring a number would not change distinct prime factors. If N^2 has 4 then N has 4

Rusty so not very sure of the answer

is it B?
1) If N already has 2 a factor then it will have 4 factors but 2 is not a factor then N would have 3 factor. So number of factors is uncertain
2) Squaring a number would not change distinct prime factors. If N^2 has 4 then N has 4

Rusty so not very sure of the answer


Correct it is B to cite an example

Stem 1 - 2N can be 2 x 105 this may have prime factors viz 2, 3, 5, 7 and N will have prime factors 3, 5, 7 OR 2N can be 2 x 210 this may have prime factors 2, 3, 5, 7 and N will have prime factors 2, 3, 5, 7. Insufficient

Stem 2 - Sufficient and reasoning as provide above

Anybody interested sharing his/her DS strategies?

aim-wsc Says
Anybody interested sharing his/her DS strategies?


@aim Keep posting questions and I am sure many would join in with their strategies and approach.

Q. The integers m and p are such that 2

1 ?

1. the greatest common factor of m and p is 2

2. the least common multiple of m and p is 30

Q. The integers m and p are such that 2

1 ?

1. the greatest common factor of m and p is 2

2. the least common multiple of m and p is 30



Is it D?
According to first statement and given data if m=n then p is also even and n+2=According to sec statement LCM of m and p is 30. Factors of 30 are 1, 2, 3, 5, 6, 10, 15, 30. m>2 So possible values of m and p are 3 and 10 or 5 and 6 and in both the cases remainder is 1.
So both statements are individually sufficient.
Is it D?
According to first statement and given data if m=n then p is also even and n+2=According to sec statement LCM of m and p is 30. Factors of 30 are 1, 2, 3, 5, 6, 10, 15, 30. m>2 So possible values of m and p are 3 and 10 or 5 and 6 and in both the cases remainder is 1.
So both statements are individually sufficient.


i concur !!
whats the rite answer
but i ve read in some books tht although its not necessary for the two answer to be the same(calculated frm the two separate options) but a question has never been cited to give two different answers.
i also faced a couple of similar questions whr i marked D as the answer but at the end found myself to be wrong...ans was either A or B
but thrs no flaw in the reasoning given above..
plz explain......

Are at least 10% of the people in Country X who are 65 years old or older employed?

1) In Country X, 11.3% of the population is 65 years old or older.

2) In Country X, of the population 65 years old or older, 20% of the men and 10% of the women are employed. x

Are at least 10% of the people in Country X who are 65 years old or older employed?

1) In Country X, 11.3% of the population is 65 years old or older.

2) In Country X, of the population 65 years old or older, 20% of the men and 10% of the women are employed. x


Its B.
Irrespective of no of men or women the percentage cannot drop below 10.
What is the number of female employees in Company
X ?



(1) If Company
X were to hire 14 more people and all of these people were females, the ratio of the number of male employees to the number of female employees would then be 16 to 9.

(2) Company

X has 105 more male employees than female employees.
is it C??

[LEFT]What is the number of female employees in Company [/LEFT]

[LEFT]X ?[/LEFT]



(1) If Company
[LEFT]X were to hire 14 more people and all of these people were females, the ratio of the number of male employees to the number of female employees would then be 16 to 9.[/LEFT]


[LEFT](2) Company

[/LEFT]

[LEFT]X has 105 more male employees than female employees.[/LEFT]


[LEFT]is it C??
[/LEFT]

I took my GMAT long time back so I dont remember option corresponds to what but the answer is : Both statements are required. Which I guess sud be C

Explaination: 1) A is not sufficient as it gives the current ratio that doesnt give us a single solution.

B) No single solution.possible.

Using Both- Current ratio- 16:9. Difference 16-9=7.

So number of female employees: [{current difference (105)/ ratio difference (7)}* Ratio of female employees (9)] 14
= [{105/7}*9]-14 = [ 15*9 ] 14 = 135-14 = 121

Hence the answer is C
c153 Says
where do we find answers to these??


Please post DS questions in the corresponding thread.

I request the Mods to move the above posts to DS section...
A jewelry dealer initially offered a bracelet for sale at an asking price that would give a profit to the dealer of 40 percent of the original cost. What was the original cost of the bracelet?
(1) After reducing this asking price by 10 percent, the jewelry dealer sold the bracelet at a profit of $403.
(2) The jewelry dealer sold the bracelet for $1,953.

can some one help me in this ;;the answer given is A but with option 2 also we can get the answer??

thanks in adv
A jewelry dealer initially offered a bracelet for sale at an asking price that would give a profit to the dealer of 40 percent of the original cost. What was the original cost of the bracelet?
(1) After reducing this asking price by 10 percent, the jewelry dealer sold the bracelet at a profit of $403.
(2) The jewelry dealer sold the bracelet for $1,953.

can some one help me in this ;;the answer given is A but with option 2 also we can get the answer??

thanks in adv


If you consider the asking price and the selling price to be the same then you can get the answer from 2nd statement also else not.
Where did u pick this question from?
If you consider the asking price and the selling price to be the same then you can get the answer from 2nd statement also else not.
Where did u pick this question from?


hi man thanks.i got this from a book called gmat plus. it is free to download as i have found on one site while searching for GMAT tips;

here is the link it consists of all the sections

My Home Page

bye